18 votos

Cómo evaluar $\int_0^1 (\arctan x)^2 \ln(\frac{1+x^2}{2x^2}) dx$

Evaluar
$$ \int_{0}^{1} \arctan^{2}\left(\, x\,\right) \ln\left(\, 1 + x^{2} \over 2x^{2}\,\right)\,{\rm d}x $$

He sustituido $x \equiv \tan\left(\,\theta\,\right)$ y consiguió

$$ -\int^{\pi/4}_{0}\theta^{2}\,{\ln\left(\, 2\sin^{2}\left(\,\theta\,\right)\,\right) \over \cos^{2}\left(\,\theta\,\right)}\,{\rm d}\theta $$

Después de esto, pensé en utilizar la expansión de Taylor de $\ln\left(\, 2\sin^{2}\left(\,\theta\,\right)\,\right)$ cerca de cero, pero eso no sirvió de nada.

Por favor, ¡ayuda!

0 votos

Se podría reducir a $\log$ integrales expandiendo $\arctan(x)$ como $\frac{i}{2}(\log(1+i x)-\log(1-i x))$ . La expresión resultante debería ser (en principio) expresable en $\log$ , $\text{Li}$ y $\zeta$ valores.

2 votos

La integral considerada es igual a $$0.119548664034710922477515058213 .$$

17voto

Godsaur Puntos 121

Resultados utilizados

Me limitaré a exponer el siguiente resultado ya que no quiero replicar el brillante trabajo de Random Variable en esta respuesta . $$\int^\frac{\pi}{4}_0\ln^2(\cos{x})\ {\rm d}x=\Im{\rm Li}_3(1-i)-\frac{\mathbf{G}}{2}\ln{2}+\frac{7\pi^3}{192}+\frac{5\pi}{16}\ln^2{2}$$ También es bastante fácil demostrar que $$\sum^\infty_{n=1}\frac{H_n}{n^2}z^n={\rm Li}_3(z)-{\rm Li}_3(1-z)+{\rm Li}_2(1-z)\ln(1-z)+\frac{1}{2}\ln{z}\ln^2(1-z)+\zeta(3)$$


Dividir la integral

Podemos dividir la integral en 3 integrales más simples. $$\mathscr{I}=-\ln{2}\underbrace{\int^1_0\arctan^2{x}\ {\rm d}x}_{\mathscr{I}_1} -2\underbrace{\int^1_0\arctan^2{x}\ln{x}\ {\rm d}x}_{\mathscr{I}_2}+\underbrace{\int^1_0\arctan^2{x}\ln(1+x^2)\ {\rm d}x}_{\mathscr{I}_3}$$


Evaluación de $\mathscr{I}_1$

Integrar por partes. \begin{align} \mathscr{I}_1 =&x\arctan^2{x}\Bigg{|}^1_0-\int^1_0\frac{2x\arctan{x}}{1+x^2}{\rm d}x\\ =&\frac{\pi^2}{16}-\left[\ln(1+x^2)\arctan{x}\right]^1_0+\int^1_0\frac{\ln(1+x^2)}{1+x^2}{\rm d}x\\ =&\frac{\pi^2}{16}-\frac{\pi}{4}\ln{2}-2\int^\frac{\pi}{4}_0\ln(\cos{x})\ {\rm d}x\\ =&\frac{\pi^2}{16}-\frac{\pi}{4}\ln{2}+\frac{\pi}{2}\ln{2}+2\sum^\infty_{n=1}\frac{(-1)^n}{n}\int^\frac{\pi}{4}_0\cos(2nx)\ {\rm d}x\\ =&\frac{\pi^2}{16}+\frac{\pi}{4}\ln{2}+\sum^\infty_{n=1}\frac{(-1)^n\sin(n\pi/2)}{n^2}\\ =&\frac{\pi^2}{16}+\frac{\pi}{4}\ln{2}+\sum^\infty_{n=0}\frac{(-1)^{2n+1}(-1)^n}{(2n+1)^2}\\ =&\frac{\pi^2}{16}+\frac{\pi}{4}\ln{2}-\mathbf{G} \end{align}


Evaluación de $\mathscr{I}_2$ $\require{cancel}$ \begin{align} \mathscr{I}_2 =&\color{red}{\cancelto{0}{\color{grey}{x\arctan^2{x}\ln{x}\Bigg{|}^1_0}}}-\int^1_0\arctan^2{x}\ {\rm d}x-\int^1_0\frac{2x\arctan{x}\ln{x}}{1+x^2}{\rm d}x\\ =&-\frac{\pi^2}{16}-\frac{\pi}{4}\ln{2}+\mathbf{G}+2\sum^\infty_{n=0}\frac{(-1)^nH_{2n+1}}{(2n+3)^2}-\sum^\infty_{n=0}\frac{(-1)^nH_{n}}{(2n+3)^2}\\ =&\frac{\pi^3}{16}-\frac{\pi^2}{16}-\frac{\pi}{4}\ln{2}+\mathbf{G}-2\sum^\infty_{n=0}\frac{(-1)^nH_{2n+1}}{(2n+1)^2}+\sum^\infty_{n=1}\frac{(-1)^{n}H_n}{(2n+1)^2} \end{align} Desde \begin{align} \sum^\infty_{n=0}\frac{(-1)^nH_{2n+1}}{(2n+1)^2} =&\Im\sum^\infty_{n=1}\frac{H_n}{n^2}i^n\\ =&-\Im{\rm Li}_3(1-i)-\frac{\mathbf{G}}{2}\ln{2}-\frac{\pi}{16}\ln^2{2} \end{align} y \begin{align} \sum^\infty_{n=1}\frac{(-1)^nH_n}{(2n+1)^2} =&\int^1_0\frac{\ln{x}\ln(1+x^2)}{1+x^2}{\rm d}x\\ =&-2\int^\frac{\pi}{4}_0\left(\ln(\sin{x})-\ln(\cos{x})\right)\ln(\cos{x})\ {\rm d}x\\ =&-\frac{1}{8}\frac{\partial^2{\rm B}}{\partial a\partial b}\left(\frac{1}{2},\frac{1}{2}\right)+2\int^\frac{\pi}{4}_0\ln^2(\cos{x})\ {\rm d}x\\ =&2\Im{\rm Li}_3(1-i)-\mathbf{G}\ln{2}+\frac{3\pi^3}{32}+\frac{\pi}{8}\ln^2{2} \end{align} Tenemos $$\mathscr{I}_2=4\Im{\rm Li}_3(1-i)+\mathbf{G}+\frac{5\pi^3}{32}+\frac{\pi}{4}\ln^2{2}-\frac{\pi^2}{16}-\frac{\pi}{4}\ln{2}$$


Evaluación de $\mathscr{I}_3$

$\mathscr{I}_3$ es bastante sencillo de evaluar. \begin{align} \mathscr{I}_3 =&x\arctan^2{x}\ln(1+x^2)\Bigg{|}^1_0-\int^1_0\frac{2x\arctan{x}\ln(1+x^2)}{1+x^2}{\rm d}x-\int^1_0\frac{2x^2\arctan^2{x}}{1+x^2}{\rm d}x\\ =&\frac{\pi^2}{16}\ln{2}-\frac{1}{2}\ln^2(1+x^2)\arctan{x}\Bigg{|}^1_0+\frac{1}{2}\int^1_0\frac{\ln^2(1+x^2)}{1+x^2}{\rm d}x-2\int^1_0\arctan^2{x}\ {\rm d}x\\&+2\int^1_0\frac{\arctan^2{x}}{1+x^2}{\rm d}x\\ =&2\mathbf{G}+\frac{\pi^3}{96}+\frac{\pi^2}{16}\ln{2}-\frac{\pi}{8}\ln^2{2}-\frac{\pi^2}{8}-\frac{\pi}{2}\ln{2}+2\int^\frac{\pi}{4}_0\ln^2(\cos{x})\ {\rm d}x\\ =&2\Im{\rm Li}_3(1-i)-\mathbf{G}\ln{2}+2\mathbf{G}+\frac{\pi^3}{12}+\frac{\pi^2}{16}\ln{2}+\frac{\pi}{2}\ln^2{2}-\frac{\pi^2}{8}-\frac{\pi}{2}\ln{2} \end{align}


La forma cerrada

Combinando estos resultados, obtenemos \begin{align} \mathscr{I}=\Large{\boxed{\displaystyle \color{red}{-6\Im{\rm Li}_3(1-i)-\frac{11\pi^3}{48}-\frac{\pi}{4}\ln^2{2}}}} \end{align} como la forma cerrada.

1 votos

@sos440 Viniendo de un experto en integración como tú, este cumplido significa mucho. Gracias.

0 votos

@M.N.C.E. ¡Gracias!

8voto

Samurai, esta es la segunda vez que publicas problemas que se relacionan conmigo. Primero publicaste esta pregunta aquí que es exactamente similar con mi problema clasificado en Brilliant.org . He planteado la objeción a los mods pero no pueden hacer nada ya que tu post no viola ninguna regla aquí. Bien, de acuerdo. Puedo aceptar sus razones. Ahora usted publica esta pregunta que creo que está tomada de uno de los problemas propuestos en Concurso de Integración Brillante - Temporada 1 que tenía en Brilliant.org. El problema original fue propuesto por Jatin Yadav como PROBLEMA 7 pero un día después borró este problema y lo cambió por otro después de que nadie pudiera resolverlo, incluido él mismo. Según él, se toma de aquí, en Matemáticas S.E. Tal vez quiera echar un vistazo allí.

He intentado solucionar este problema durante horas pero sin éxito. Aquí está mi intento:

Establecer $x=\tan y$ obtenemos \begin{align} I&=\int_0^1\arctan^2x\,\ln\left(\frac{1+x^2}{2x^2}\right)\,dx\\ &=-\int_0^{\pi/4} \frac{y^2\ln\left(2\sin^2y\right)}{\cos^2y}\,dy\\ &=-2\int_0^{\pi/4} \frac{y^2\ln\left(1-\cos2y\right)}{1+\cos2y}\,dy\\ &=-\frac{1}{4}\int_0^{\pi/2} \frac{t^2\ln\left(1-\cos t\right)}{1+\cos t}\,dt\qquad\Rightarrow\qquad t=2y\\ \end{align}

Utilice la integración por partes tomando $u=t^2$ y $dv=\dfrac{\ln\left(1-\cos t\right)}{1+\cos t}\,dt$ entonces \begin{align} v&=\int\frac{\ln\left(1-\cos t\right)}{1+\cos t}\,dt \end{align} Utilice la integración por partes tomando $u=\ln\left(1-\cos t\right)$ y $dv=\dfrac{dt}{1+\cos t}$ , por Sustitución de Weierstrass : $x=\tan\left(\dfrac{t}{2}\right)$ entonces \begin{align} v=\int\frac{dt}{1+\cos t}=\int \,dx=\tan\left(\frac{t}{2}\right)=\frac{\sin t}{1+\cos t} \end{align} Por lo tanto, \begin{align} \int\frac{\ln\left(1-\cos t\right)}{1+\cos t}\,dt &=\frac{\sin t}{1+\cos t}\ln\left(1-\cos t\right)-\int\frac{\sin t}{1+\cos t}\cdot\frac{\sin t}{1-\cos t}\,dt\\ &=\frac{\sin t}{1+\cos t}\ln\left(1-\cos t\right)-t \end{align} y \begin{align} I&=-\frac{1}{4}\left[\frac{t^2\sin t}{1+\cos t}\ln\left(1-\cos t\right)-t^3\right]_0^{\pi/2}+\frac{1}{2}\int_0^{\pi/2}\left[\frac{t\sin t}{1+\cos t}\ln\left(1-\cos t\right)-t^2\right]\,dt\\ &=\frac{\pi^3}{32}+\frac{1}{2}\int_0^{\pi/2}\frac{t\sin t}{1+\cos t}\ln\left(1-\cos t\right)\,dt-\frac{\pi^3}{48}\\ &=\frac{\pi^3}{96}+\frac{1}{2}\int_0^{\pi/2}\frac{t\sin t}{1+\cos t}\ln\left(1-\cos t\right)\,dt \end{align} Considere \begin{equation} I(a)=\int_0^{\pi/2} \frac{t\sin t}{1+\cos t}\ln\left(1-a\cos t\right)\,dt \end{equation} para que $I(0)=0$ y \begin{align} I'(a)&=-\int_0^{\pi/2} \frac{t\sin t\cos t}{(1-a\cos t)(1+\cos t)}\,dt\\ &=\frac{1}{1+a}\int_0^{\pi/2} \left(\frac{t\sin t}{1+\cos t}-\frac{t\sin t}{1-a\cos t}\right)\,dt\\ \end{align} Ahora considere \begin{equation} I(b)=\int_0^{\pi/2} \frac{t\sin t}{1+b\cos t}\,dt \end{equation} Utilice la integración por partes tomando $u=t$ y $dv=\dfrac{\sin t}{1+b\cos t}\,dt$ entonces \begin{align} I(b)&= \frac{t\ln(1+b\cos t)}{b}\bigg|_0^{\pi/2}-\frac{1}{b}\int_0^{\pi/2}\ln(1+b\cos t)\,dt\\ &=-\frac{1}{b}\int_0^{\pi/2}\ln(1+b\cos t)\,dt \end{align} Considere \begin{equation} J(b)=\int_0^{\pi/2}\ln(1+b\cos t)\,dt \end{equation} para que $J(0)=0$ y \begin{align} J'(b)&=\int_0^{\pi/2} \frac{\cos t}{1+b\cos t}\,dt\\ &=\frac{1}{b}\int_0^{\pi/2} \left(1-\frac{1}{1+b\cos t}\right)\,dt\\ &=\frac{\pi}{2b}-\int_0^{\pi/2} \frac{dt}{1+b\cos t}\qquad\Rightarrow\qquad x=\tan\left(\frac{t}{2}\right)\\ &=\frac{\pi}{2b}-\int_0^{1} \frac{2}{1+b+(1-b)x^2}\,dx\qquad\Rightarrow\qquad x=\sqrt{\frac{1+b}{1-b}}\tan z\\ &=\frac{\pi}{2b}-\frac{2}{\sqrt{1-b^2}}\arctan\left(\sqrt{\frac{1-b}{1+b}}\right)\\ J(b)&=\frac{\pi}{2}\ln b-\int\frac{2}{\sqrt{1-b^2}}\arctan\left(\sqrt{\frac{1-b}{1+b}}\right)\,db\\ \end{align} De nuevo utilizamos la integración por partes tomando $u=\arctan\left(\sqrt{\frac{1-b}{1+b}}\right)$ y $dv=\dfrac{2}{\sqrt{1-b^2}}$ tenemos \begin{align} J(b)&=\frac{\pi}{2}\ln b-2\arctan\left(\sqrt{\frac{1-b}{1+b}}\right)\arcsin b-\int\frac{\arcsin b}{1-b}\sqrt{\frac{1-b}{1+b}}\,db\\ &=\frac{\pi}{2}\ln b-2\arctan\left(\sqrt{\frac{1-b}{1+b}}\right)\arcsin b-\int\frac{\arcsin b}{\sqrt{1-b^2}}\,db\\ &=\frac{\pi}{2}\ln b-2\arctan\left(\sqrt{\frac{1-b}{1+b}}\right)\arcsin b-\frac{\arcsin^2 b}{2} \end{align} Por lo tanto, \begin{align} I'(a)&=\frac{\pi^2}{8(1+a)}-\frac{1}{a(1+a)}\left[\frac{\pi}{2}\ln (-a)-2\arctan\left(\sqrt{\frac{1+a}{1-a}}\right)\arcsin (-a)-\frac{\arcsin^2(-a)}{2}\right]\\ \end{align} A partir de este paso, me rindo. Tal vez alguien más quiera continuarlo. Sea mi invitado...

0 votos

Lo siento, pero el último resultado no parece correcto. Queremos calcular $I(1) = \int_0^1 da I'(a)$ pero esta integral diverge debido a la $\ln(-a)/(a(1+a))$ plazo.

0 votos

@user111187 Por eso he dicho, "Me rindo" . Estoy demasiado cansado. Lo siento...

0 votos

¿Anastasiya-Romanova? Me he topado con esta integral y me he dado cuenta de que su respuesta es muy antigua, así que quizá ya no esté en ello. Pero creo que se podría resolver la integral indefinida en términos de polilogaritmos comprando sustituyendo $\omega = \frac{1+ix}{2}$ . De esta manera se consigue $\arctan x = (\log \omega + \mbox{Li}_1(\omega))/2i$ . El factor logarítmico de la función integrante también puede escribirse fácilmente en términos de $\omega$ ...

7voto

psychotik Puntos 171

Esto no es una respuesta, pero mi enfoque sugiere que la respuesta es

$$ I := \int_{0}^{1} \arctan^{2} x \log \left( \frac{x^{2}+1}{2x^{2}} \right) \, dx = \frac{19\pi^{3}}{192} + \frac{5\pi}{16}\log^{2}2 - 6 \Im \mathrm{Li}_{3}\left( \frac{1+i}{2} \right). $$


Mi enfoque es escribir

$$ I = \int_{0}^{1} \arctan^{2} x \log \left( \frac{x^{2}+1}{2} \right) \, dx - 2\int_{0}^{1} \arctan^{2} x \log x \, dx, $$

introducir la función $f(z) = \log \left( \frac{1+iz}{\sqrt{2}} \right)$ y escribir

$$ \arctan^{2} x \log \left( \frac{x^{2}+1}{2} \right) = -\frac{1}{4} ( f(x) - f(-x))^{2}(f(x) + f(-x)). $$

Esto permite escribir, con un poco de ayuda del análisis complejo,

$$ \int_{0}^{1} \arctan^{2} x \log \left( \frac{x^{2}+1}{2} \right) \, dx = - \frac{\sqrt{2}}{4} \int_{-\pi/4}^{\pi/4} \theta^{2}\log(\sqrt{2}e^{i\theta} - 1) e^{i\theta} \, d\theta, $$

que parece más manejable que el original. Pero me quedé atascado aquí.

0 votos

Así que, $\mathrm{Im} \; \mathrm{Li}_3\left((1+i)/2\right)$ Nos volvemos a encontrar.

0 votos

@Gahawar, ¿has conocido a este tipo incómodo $\Im \mathrm{Li}_{3}(\sqrt{i/2})$ ¿en algún lugar antes?

0 votos

Sí, hace unos meses. No fui capaz de encontrar una forma cerrada para ello, sin embargo Cleo encontró una expresión que implica una serie hipergeométrica desagradable. math.stackexchange.com/questions/918680/

5voto

Vladimir Reshetnikov Puntos 18017

Por cierto, existe una antiderivada de forma cerrada (que podría demostrarse por diferenciación): $$\int\arctan^2x\cdot \ln\left(\frac{1+x^2}{2x^2}\right)\,dx=\\ \frac16\left[3 i \left\{\left(2 \operatorname{Li}_2(i x)-2 \operatorname{Li}_2(-i x)+\operatorname{Li}_2\left(\frac{2 x}{x+i}\right)-\operatorname{Li}_2\left(\frac{2 x}{x-i}\right)\right)\cdot \ln \left(\frac{1+x^2}{x^2}\right)\\ +2 \left(2 \operatorname{Li}_3\left(\frac{x}{x-i}\right)-2 \operatorname{Li}_3\left(\frac{x}{x+i}\right)+\operatorname{Li}_3\left(\frac{2 x}{x+i}\right)-\operatorname{Li}_3\left(\frac{2 x}{x-i}\right)\right)\\ +\left(\operatorname{Li}_2\left(\frac{1}{2}-\frac{i x}{2}\right)-\operatorname{Li}_2\left(\frac{i x}{2}+\frac{1}{2}\right)\right)\cdot\ln2\right\}\\ +3 \left(2 \operatorname{Li}_2\left(-x^2\right)+\ln ^2\left(1+x^2\right)+\ln \left(1+x^2\right)\cdot\ln2-2 \ln ^22\right)\cdot\arctan x\\ +6 x \ln \left(\frac{1+x^2}{2 x^2}\right)\cdot\arctan^2x+4 \arctan^3x\right]\color{gray}{+C}$$ Esto nos permite evaluar una integral definida sobre cualquier región.

0 votos

Este resultado se puede obtener con Mathematica o WolframAlpha evaluando Integrate[ (1/2 I Log[1 - I x] - 1/2 I Log[1 + I x])^2 (Log[1 + I x] + Log[1 - I x] - Log[2] - 2 Log[x]), x] y la posterior simplificación.

0 votos

(+1) ¡Maravilloso! La solución más potente para esta cuestión.

3voto

FDP Puntos 448

No tengo (todavía) una solución completa por ahora.

Dejemos que $I=\displaystyle \int_0^1 (\arctan(x))^2 \ln\Big(\dfrac{1+x^2}{2x^2}\Big)dx$

$I=\displaystyle \int_0^1 (\arctan(x))^2\ln(1+x^2)dx-2\int_0^1 (\arctan(x))^2\ln(x)dx-\ln(2)\int_0^1 (\arctan(x))^2dx$

Sé cómo calcular la última ;)

Dejemos que $J=\displaystyle \int_0^{\frac{\pi}{2}} \log(\sin x)dx$

El valor de $J$ es bien conocido por ser $-\dfrac{\pi}{2}\ln 2$

Realizar la integración por partes:

$J=\displaystyle \Big[x\log(\sin x)\Big]_0^{\frac{\pi}{2}}-\int_0^{\frac{\pi}{2}}\dfrac{x}{\tan x}dx=-\int_0^{\frac{\pi}{2}}\dfrac{x}{\tan x}dx$

Dejemos que $K=-J$

Realizar el cambio de variable $u=\tan x$

$K=\displaystyle \int_0^{+\infty}\dfrac{\arctan x}{x(1+x^2)}dx$

$K\displaystyle =\int_0^{1}\dfrac{\arctan x}{x(1+x^2)}dx+\int_1^{+\infty}\dfrac{\arctan x}{x(1+x^2)}dx$

En la segunda integral, en el miembro derecho realizar el cambio de variable $u=\dfrac{1}{x}$ :

$K= \displaystyle\int_0^1 \dfrac{\arctan x }{x(1+x^2)}dx+ \int_0^1 \dfrac{x\arctan \Big(\dfrac{1}{x}\Big) }{1+x^2}dx$

Para $x>0$ , $\arctan \Big(\dfrac{1}{x}\Big)+\arctan x=\dfrac{\pi}{2}$

Por lo tanto:

$K=\displaystyle\int_0^1 \dfrac{\arctan x }{x(1+x^2)}dx+\dfrac{\pi}{2}\int_0^1\dfrac{x}{1+x^2}dx-\int_0^1\dfrac{x\arctan x}{1+x^2}dx$

Para $x\neq 0$ , $\dfrac{1}{x(1+x^2)}=\dfrac{1}{x}-\dfrac{x}{1+x^2}$

$K=\displaystyle\int_0^1 \dfrac{\arctan x }{x}dx-2\int_0^1 \dfrac{x\arctan x }{1+x^2}dx+\dfrac{\pi}{2}\int_0^1\dfrac{x}{1+x^2}dx$

La derivada de $(\arctan x)^2$ es $\dfrac{2\arctan x}{1+x^2}$

Por lo tanto:

$\displaystyle \int_0^1 \dfrac{2x\arctan x }{1+x^2}dx=\Big[x(\arctan x)^2\Big]_0^1-\int_0^1 (\arctan x)^2dx=\dfrac{\pi^2}{16}-\int_0^1 (\arctan x)^2dx$

Por lo tanto:

$K=\displaystyle\int_0^1 \dfrac{\arctan x }{x}dx-\dfrac{\pi^2}{16}+\int_0^1 (\arctan x)^2dx+\dfrac{\pi}{4}\Big[\log(1+x^2)\Big]_0^1$

Recall $K=\dfrac{\pi}{2}\ln 2$

Por lo tanto:

$\displaystyle\int_0^1 (\arctan x)^2 dx=\dfrac{\pi^2}{16}-G+\dfrac{\pi}{4}\ln 2$

Dónde $G=\displaystyle\int_0^1 \dfrac{\arctan x }{x}dx$ es la constante del catalán.

i-Ciencias.com

I-Ciencias es una comunidad de estudiantes y amantes de la ciencia en la que puedes resolver tus problemas y dudas.
Puedes consultar las preguntas de otros usuarios, hacer tus propias preguntas o resolver las de los demás.

Powered by:

X